Định lý thặng dư trung hoa và ứng dụng

11 18 1
Định lý thặng dư trung hoa và ứng dụng

Đang tải... (xem toàn văn)

Tài liệu hạn chế xem trước, để xem đầy đủ mời bạn chọn Tải xuống

Thông tin tài liệu

ĐỊNH LÝ THẶNG DƯ TRUNG HOA VÀ ỨNG DỤNG (Chinese Remainder Theorem and its application) Hàn Tín danh tướng nhà Hán người đời sau ca tụng Binh Tiên cầm quân bách chiến bách thắng, thiên hạ vơ địch, thiên cổ khơng có người thứ hai Tương truyền rằng, Hàn Tín kiểm điểm quân số (một việc mà tưởng chừng dễ dàng lại khơng biết phải đếm mà chưa có phân thống kê, xử lý số liệu bây giờ) thường làm sau Ơng cho qn lính xếp thành hàng 3, hàng 5, hàng Mỗi lần vậy, quân lính báo cho Hàn Tín số người hàng cuối (có thể khơng đủ 3, hay 7) Từ đó, ơng ta suy số qn xác Cách điểm số qn ơng thể qua thơ sau: “Tam nhân đồng hành thất thập hy Ngũ thụ mai hoa thấp Thất tử đồn viên bán nguyệt Trừ bách linh ngũ tiện đắc chi” Thực chất toán Hàn Tín điểm binh việc giải hệ phương trình đồng dư tuyến tính theo modulo 3, Thuật tốn mà ơng dùng dựa định lý tiếng toán học: Định lý thặng dư Trung Hoa (Chinese Remainder Theorem) *Định lý thặng dư Trung Hoa: Cho 𝑚 , 𝑚 ,…, 𝑚 số nguyên dương đôi nguyên tố nhau, 𝑎 , 𝑎 ,…, 𝑎 số nguyên Khi hệ phương trình đồng dư tuyến tính 𝑥 ≡ 𝑎 (𝑚𝑜𝑑 𝑚 ) 𝑥 ≡ 𝑎 (𝑚𝑜𝑑 𝑚 ) … 𝑥 ≡ 𝑎 (𝑚𝑜𝑑 𝑚 ) có nghiệm Nếu 𝑥 số nghiệm, 𝑥 thoả hệ phương trình đồng dư 𝑥 có dạng 𝑥 + 𝑘𝑚 với số nguyên 𝑘 đó, 𝑚 = 𝑚 𝑚 …𝑚 Chứng minh Dễ thấy 𝑚/𝑚 , 𝑚 = nên tồn số nguyên 𝑏 cho 𝑚/𝑚 𝑏 ≡ (𝑚𝑜𝑑 𝑚 ), đồng thời 𝑚/𝑚 𝑏 ≡ (𝑚𝑜𝑑 𝑚 ) ∀𝑖 ≠ 𝑗 Đặt 𝑥 = 𝑚 𝑏𝑎 𝑚 Ta có: 𝑥 ≡ 𝑚 𝑏 𝑎 ≡ 𝑎 (𝑚𝑜𝑑 𝑚 ) 𝑚 Điều nghĩa 𝑥 nghiệm hệ phương trình cho Nếu 𝑥 𝑥 hai nghiệm hệ phương trình đồng dư, 𝑥 ≡ 𝑥 (𝑚𝑜𝑑 𝑚 ) với 𝑖 = 1, 𝑟 𝑥 ≡ 𝑥 (𝑚𝑜𝑑 𝑚) Chứng minh hoàn tất Định lý thặng dư Trung Hoa phần quan trọng lý thuyết số kết hữu ích cho phép người ta tìm nghiệm cho hệ phương trình đồng dư tuyến tính có modulo cặp số ngun đơi ngun tố Đồng thời cơng cụ mạnh tốn, vấn đề mang tính xây dựng Ví dụ Tìm nghiệm hệ phương trình đồng dư tuyến tính sau 𝑥 ≡ (𝑚𝑜𝑑 3) 𝑥 ≡ (𝑚𝑜𝑑 4) 𝑥 ≡ (𝑚𝑜𝑑 5) Lời giải Ta tìm nghiệm tốn dựa chứng minh định lý thặng dư Trung Hoa Trong trường hợp 𝑚 = 3.4.5 = 60 Ta có nghiệm hệ 𝑥 = 20𝑏 𝑎 + 15𝑏 𝑎 + 12𝑏 𝑎 = 40𝑏 + 15𝑏 + 36𝑏 Với 𝑏 , 𝑏 , 𝑏 xác định sau 60 ⎧ 𝑏 ≡ (𝑚𝑜𝑑 3) ⎪3 60 𝑏 ≡ (𝑚𝑜𝑑 4) ⎨4 ⎪60 ⎩ 𝑏 ≡ (𝑚𝑜𝑑 5) 2𝑏 ≡ (𝑚𝑜𝑑 3) ⇒ 3𝑏 ≡ (𝑚𝑜𝑑 4) 2𝑏 ≡ (𝑚𝑜𝑑 5) Ta chọn 𝑏 = 2, 𝑏 = 3, 𝑏 = Khi 𝑥 = 40.2 + 15.3 + 36.3 = 233 ≡ 53 (𝑚𝑜𝑑 60) Vậy nghiệm hệ phương trình đồng dư 𝑥 = 53 + 60𝑘, 𝑘 ∈ ℤ Ví dụ Giải phương trình đồng dư 𝑥 ≡ (𝑚𝑜𝑑 144) Lời giải Ta có 144 = 9.16 (9,16) = nên theo định lý thặng dư Trung Hoa nghiệm tốn nghiệm hệ phương trình đồng dư tuyến tính 𝑥 ≡ (𝑚𝑜𝑑 9) 𝑥 ≡ (𝑚𝑜𝑑 16) Phương trình 𝑥 ≡ (𝑚𝑜𝑑 16) có nghiệm 𝑥 ≡ ±1, ±7 (𝑚𝑜𝑑 16) Phương trình 𝑥 ≡ (𝑚𝑜𝑑 9) có nghiệm 𝑥 ≡ ±1 (𝑚𝑜𝑑 9) Quá trình xử lý dành cho bạn đọc Các nghiệm phương trình 𝑥 ≡ ±1, ±17, ±55, ±71 (𝑚𝑜𝑑 144) Nhận xét: Ví dụ toán nhất, thể rõ ràng ý tưởng việc ứng dụng định lý thặng dư Trung Hoa để tìm nghiệm phương trình đồng dư hay hệ phương trình đồng dư Ví dụ (IMO 1989) Chứng minh với số nguyên dương 𝑛, tồn 𝑛 số tự nhiên liên tiếp mà 𝑛 số luỹ thừa số nguyên tố Lời giải Với 𝑛 ∈ ℤ , xét 𝑛 số nguyên tố phân biệt 𝑝 , 𝑝 , … , 𝑝 Xét hệ phương trình đồng dư 𝑥 ≡ 𝑝 − (𝑚𝑜𝑑 𝑝 ) 𝑥 ≡ 𝑝 − (𝑚𝑜𝑑 𝑝 ) … ⎨ ⎩𝑥 ≡ 𝑝 − (𝑚𝑜𝑑 𝑝 ) ⎧ Theo định lý thặng dư Trung Hoa hệ phương trình có nghiệm ⇔ ∃𝑎 ∈ ℤ: 𝑎 ≡ 𝑝 − 𝑚𝑜𝑑 𝑝 ∀𝑖 = 1, 𝑛 Từ suy số 𝑎 + 1, 𝑎 + 2, … , 𝑎 + 𝑛 luỹ thừa với số mũ nguyên dương số nguyên tố Ví dụ (VMO 2008) Cho 𝑚 = 2007 Hỏi có số nguyên dương 𝑛 ≤ 𝑚 thoả mãn điều kiện 𝑛(2𝑛 + 1)(5𝑛 + 2) ⋮ 𝑚 Lời giải Ta có 𝑚 = 223 =3 223 =𝑛 𝑛 Do (𝑚, 10) = ⇒ 𝑛(2𝑛 + 1)(5𝑛 + 2) ⋮ 𝑚 ⇔ 𝑚|10.5.2𝑛(2𝑛 + 1)(5𝑛 + 2) = 10𝑛(10𝑛 + 5)(10𝑛 + 4) ⇔ 𝑚|𝑥(𝑥 + 2)(𝑥 + 4) 𝑥 = 10𝑛 Từ ta có hệ phương trình đồng dư sau: 𝑥 ≡ (𝑚𝑜𝑑 10) 𝑥(𝑥 + 5)(𝑥 + 4) ≡ (𝑚𝑜𝑑 𝑛 ) 𝑥(𝑥 + 5)(𝑥 + 4) ≡ (𝑚𝑜𝑑 𝑛 ) Vì khơng ước chung 𝑥, 𝑥 + 4, 𝑥 + nên 𝑥(𝑥 + 5)(𝑥 + 4) ≡ (𝑚𝑜𝑑 𝑛 ) 𝑥 ≡ 𝑟 (𝑚𝑜𝑑 𝑛 ) 𝑟 ∈ {0, −4, −5} Tương tự 𝑥(𝑥 + 5)(𝑥 + 4) ≡ (𝑚𝑜𝑑 𝑛 ) 𝑥 ≡ 𝑟 (𝑚𝑜𝑑 𝑛 ) 𝑟 ∈ {0, −4, −5} Vậy 𝑚|𝑛(2𝑛 + 5)(5𝑛 + 4) ⇔ 𝑥 ≡ (𝑚𝑜𝑑 10), 𝑥 ≡ 𝑟 (𝑚𝑜𝑑 𝑛 ), 𝑥 ≡ 𝑟 (𝑚𝑜𝑑 𝑛 ) Vậy với 𝑛 ≤ 𝑚 thoả mãn điều kiện số số 𝑥 ≤ 10𝑛 𝑛 thoả mãn 𝑥 ≡ (𝑚𝑜𝑑 10), 𝑥 ≡ 𝑟 (𝑚𝑜𝑑 𝑛 ), 𝑥 ≡ 𝑟 (𝑚𝑜𝑑 𝑛 ) Với cách chọn 𝑟 ∈ {0, −4, −5} 𝑟 ∈ {0, −4, −5}, theo định lý thặng dư Trung Hoa ta có số 𝑥 ≤ 10𝑛 𝑛 thoả mãn 𝑥 ≡ (𝑚𝑜𝑑 10), 𝑥 ≡ 𝑟 (𝑚𝑜𝑑 𝑛 ), 𝑥 ≡ 𝑟 (𝑚𝑜𝑑 𝑛 ) Vậy có số thoả mãn điều kiện đề *Tổng quát toán: Cho 𝑚 = 𝑝 𝑝 … 𝑝 (𝑝 ∈ ℘, 𝛼 ∈ ℕ) 𝑓(𝑥) đa thức với hệ số ngun Khi phương trình đồng dư 𝑓(𝑥) ≡ (𝑚𝑜𝑑 𝑚) có nghiệm tất phương trình đồng dư 𝑓(𝑥) ≡ 𝑚𝑜𝑑 𝑝 , 𝑖 = 1, 𝑘 có nghiệm Nếu gọi số nghiệm phương trình (𝑥) ≡ 𝑚𝑜𝑑 𝑝 , 𝑖 = 1, 𝑘 𝑛 phương trình (𝑥) ≡ (𝑚𝑜𝑑 𝑚) có 𝑛 𝑛 … 𝑛 nghiệm modulo 𝑚 Ví dụ (Đài Loan MO 2002) Một điểm nguyên 𝑋 mặt phẳng gọi nhìn thấy từ 𝑂 đoạn thẳng 𝑂𝑋 khơng chứa điểm ngun khác ngồi 𝑂 𝑋 Chứng minh với số nguyên dương 𝑛, tồn hình vng 𝑛 điểm ngun (các cạnh song song với trục toạ độ) cho khơng có điểm bên hình vng nhìn thấy từ gốc toạ độ Lời giải Giả sử điểm thấp phía bên trái hình vng có toạ độ (𝑥 , 𝑦 ) Ta chứng minh chọn (𝑥 , 𝑦 ) cho hình vng với điểm ngun nhận (𝑥 , 𝑦 ) làm góc 𝑛 điểm ngun cạnh khơng thể nhìn thấy từ gốc toạ độ Điều đạt cách đảm bảo toạ độ điểm chia hết cho số số nguyên tố đó; điều nghĩa cách chia tung độ hoành độ cho số nguyên tố đó, ta tìm điểm ngun khác nằm gốc toạ độ điểm Hơn nữa, ta thấy điểm 𝑋 = (𝑥, 𝑦) nhìn thấy từ gốc gcd(𝑥, 𝑦) = Chọn 𝑛 số nguyên tố 𝑝 , , ≤ 𝑖, 𝑗 ≤ 𝑛, ta cần tìm 𝑥 thoả mãn hệ phương trình đồng dư ⎧ ⎪ ⎨ ⎪𝑥 ⎩ 𝑥 ≡ 𝑚𝑜𝑑 𝑝 , 𝑝 , …𝑝 , 𝑥 + ≡ 𝑚𝑜𝑑 𝑝 , 𝑝 , … 𝑝 , … + 𝑛 − ≡ 𝑚𝑜𝑑 𝑝 , 𝑝 , … 𝑝 , Tương tự, ta cần chọn 𝑦 thoả mãn hệ phương trình đồng dư ⎧ ⎪ ⎨ ⎪𝑦 ⎩ 𝑦 ≡ 𝑚𝑜𝑑 𝑝 , 𝑝 , …𝑝 , 𝑦 + ≡ 𝑚𝑜𝑑 𝑝 , 𝑝 , … 𝑝 , … + 𝑛 − ≡ 𝑚𝑜𝑑 𝑝 , 𝑝 , … 𝑝 , Cả hai giá trị tồn theo định lý thặng dư Trung Hoa Ta dễ thấy: (𝑥 + 𝑖, 𝑦 + 𝑗) ⋮ 𝑝 , ⇒ 𝑔𝑐𝑑(𝑥 + 𝑖, 𝑦 + 𝑗) > ∀0 ≤ 𝑖, 𝑗 ≤ 𝑛 Điều chứng tỏ điểm nằm bên rìa hình vng 𝑛 điểm ngun có điểm thấp bên trái (𝑥 , 𝑦 ) khơng nhìn thấy từ điểm 𝑂 gốc toạ độ Ví dụ Cho 𝑃(𝑥) đa thức với hệ số nguyên Giả sử có tập hữu hạn số nguyên tố 𝐴 = {𝑝 , 𝑝 , … , 𝑝 }, cho với số nguyên 𝑎 tồn số 𝑝 ∈ 𝐴, (𝑖 = 1, 𝑘) cho 𝑃(𝑎) ⋮ 𝑝 Chứng minh tồn số nguyên tố 𝑝 ∈ 𝐴 cho 𝑃(𝑥) chia hết cho 𝑝 với số nguyên 𝑥 Lời giải Ta có kết quen thuộc: Với 𝑥, 𝑦 ∈ ℤ, 𝑥 ≠ 𝑦, 𝑃(𝑥) ∈ ℤ[𝑥] ta có 𝑃(𝑥) − 𝑃(𝑦) ⋮ 𝑎 − 𝑏 ⇒ Nếu 𝑎 ≡ 𝑏 (𝑚𝑜𝑑 𝑝) 𝑃(𝑎) ≡ 𝑃(𝑏) (𝑚𝑜𝑑 𝑝) Ta dễ dàng chứng minh 𝑃(𝑎) ≡ 𝑃(𝑏) (𝑚𝑜𝑑 𝑝) 𝑎 ≡ 𝑏 (𝑚𝑜𝑑 𝑝) ⇒ 𝑎 ≡ 𝑏 (𝑚𝑜𝑑 𝑝) ⇔ 𝑃(𝑎) ≡ 𝑃(𝑏) (𝑚𝑜𝑑 𝑝) (∗) Giả sử không tồn 𝑝 ∈ 𝐴 cho 𝑝|𝑃(𝑛) ∀𝑛 ∈ ℤ ⇒ ∃𝑎 , 𝑎 , … , 𝑎 ∈ ℤ phân biệt cho 𝑃(𝑎 ) ≡ 𝑎 (𝑚𝑜𝑑 𝑝 ), 𝑝 ∤ 𝑎 𝑃(𝑎 ) ≡ 𝑎 (𝑚𝑜𝑑 𝑝 ), 𝑝 ∤ 𝑎 … 𝑃(𝑎 ) ≡ 𝑎 (𝑚𝑜𝑑 𝑝 ), 𝑝 ∤ 𝑎 Xét hệ phương trình đồng dư 𝑥 ≡ 𝑎 (𝑚𝑜𝑑 𝑝 ) 𝑥 ≡ 𝑎 (𝑚𝑜𝑑 𝑝 ) … 𝑥 ≡ 𝑎 (𝑚𝑜𝑑 𝑝 ) Theo định lý thặng dư Trung Hoa hệ phương trình có nghiệm 𝑥 , kết hợp với (∗) ta có: 𝑃(𝑥 ) ≡ 𝑃(𝑎 ) ≡ 𝑎 (𝑚𝑜𝑑 𝑝 ) 𝑃(𝑥 ) ≡ 𝑃(𝑎 ) ≡ 𝑎 (𝑚𝑜𝑑 𝑝 ) … 𝑃(𝑥 ) ≡ 𝑃(𝑎 ) ≡ 𝑎 (𝑚𝑜𝑑 𝑝 ) Nghĩa ∄𝑝 ∈ 𝐴, (𝑖 = 1, 𝑘): 𝑝 |𝑃(𝑥 ) nên mâu thuẫn với giả thiết Vậy điều giả sử sai, ta có điều phải chứng minh Ví dụ (HSG Trại hè Hùng Vương 2014) Chứng minh tồn 16 số nguyên dương liên tiếp cho khơng có số 16 số biểu diễn dạng |7𝑥 + 9𝑥𝑦 − 5𝑦 |, (𝑥, 𝑦 ∈ ℕ) Lời giải Đặt |7𝑥 + 9𝑥𝑦 − 5𝑦 | = 𝐴 ⇒ 28𝐴 = |(14𝑥 + 9𝑦) − 13.17𝑦 | Ta xét số dư chia 𝐴 cho 9, 13, 17 thu - 𝐴 chia cho khơng có số dư 3, - 𝐴 chia cho 13 khơng có số dư 1, 3, 4, 9, 10, 12 - 𝐴 chia cho 17 khơng có số dư 1, 2, 4, 8, 9, 13, 15, 16 Theo định lý thặng dư Trung Hoa tồn số nguyên dương 𝑛 thoả mãn 𝑛 ≡ −4 (𝑚𝑜𝑑 9) 𝑛 ≡ −2 (𝑚𝑜𝑑 13) 𝑛 ≡ (𝑚𝑜𝑑 17) Rõ ràng 𝑛 + 1, 𝑛 + 2, … , 𝑛 + 16 thoả mãn u cầu tốn Vậy ta có điều phải chứng minh Nhận xét: Từ trường hợp sở cho số nguyên dương 9, 13, 17 ta xây dựng nên hệ phương trình đồng dư tuyến tính tối ưu để 16 số nguyên dương liên tiếp khơng có dạng mà đề cho, việc làm cần có nhạy cảm tinh tế Ví dụ Chứng minh phương trình 𝑥 − 34𝑦 ≡ −1 (𝑚𝑜𝑑 𝑚) có nghiệm với 𝑚 ∈ ℤ Lời giải - Trường hợp 1: (𝑚, 3) = ⇒ 𝑥 − 34𝑦 ≡ −1 (𝑚𝑜𝑑 𝑚) ⇔ (𝑥 − 5𝑦)(𝑥 + 5𝑦) ≡ (3𝑦 + 1)(3𝑦 − 1) (𝑚𝑜𝑑 𝑚) Tập hợp số {3𝑦 − 1,3𝑦 + 1} chạy qua số không chia hết cho ⇒ ∃𝑦 ∈ ℤ: (3𝑦 + 1)(3𝑦 − 1) ⋮ 𝑚 Chọn 𝑥 = 5𝑦 ⇒ (𝑥 , 𝑦 ) cần tìm - Trường hợp 2: (𝑚, 5) = ⇒ 𝑥 − 34𝑦 ≡ −1 (𝑚𝑜𝑑 𝑚) ⇔ (𝑥 − 3𝑦)(𝑥 + 3𝑦) ≡ (5𝑦 + 1)(5𝑦 − 1) (𝑚𝑜𝑑 𝑚) Tập hợp số {5𝑦 + 1,5𝑦 − 1} chạy qua số không chia hết cho ⇒ ∃𝑦 ∈ ℤ: (5𝑦 + 1)(5𝑦 − 1) ⋮ 𝑚 Chọn 𝑥 = 3𝑦 ⇒ (𝑥 , 𝑦 ) cần tìm - Trường hợp 3: (𝑚, 3) = (𝑚, 5) ≠ Đặt 𝑚 = 𝑚 𝑚 với 𝑚 = (𝛼 ∈ ℕ∗ ), 𝑚 ∈ ℕ∗ : (𝑚 , 𝑚 ) = 1, (𝑚 , 5) = (3, 𝑚 ) = ⇒ ∃(𝑥 , 𝑦 ) ∈ (ℤ ) : 𝑥 − 34𝑦 ≡ −1 (𝑚𝑜𝑑 𝑚 ) (5, 𝑚 ) = ⇒ ∃(𝑥 , 𝑦 ) ∈ (ℤ ) : 𝑥 − 34𝑦 ≡ −1 (𝑚𝑜𝑑 𝑚 ) Từ theo định lý thặng dư Trung Hoa tồn (𝑥, 𝑦) ∈ (ℤ ) cho 𝑥 ≡ 𝑥 (𝑚𝑜𝑑 𝑚 ) 𝑥 ≡ 𝑥 (𝑚𝑜𝑑 𝑚 ) 𝑦 ≡ 𝑦 (𝑚𝑜𝑑 𝑚 ) 𝑦 ≡ 𝑦 (𝑚𝑜𝑑 𝑚 ) Vậy ta có điều phải chứng minh Ví dụ Tìm tất 𝑎, 𝑏, 𝑐 ∈ ℤ, 𝑐 ≥ cho (𝑎 + )|(𝑏 + 𝑐) với số nguyên dương 𝑛, đồng thời 2𝑎𝑏 khơng số phương Nhận xét: Đây ví dụ có độ khó tương đối cao, u cầu phải kết hợp đồng dư, thặng dư bình phương định lý thặng dư Trung Hoa để giải Sau lời giải tham khảo cho toán Lời giải Ta có (𝑎 + )|(𝑏 + 𝑐) ⇔ 𝑏 ≡ −𝑐 (𝑚𝑜𝑑 𝑎 + 𝑏 ) ⇒ 𝑏 ) (𝑎 + )|(𝑏 Mà (𝑎 + )|𝑎 + 𝑐) ⇔ 𝑏 +2 + 𝑐 ≡ (𝑚𝑜𝑑 𝑎 ⇒𝑏 + 𝑐 ≡ (𝑚𝑜𝑑 𝑎 + +2 ) + 𝑐 ≡ (𝑚𝑜𝑑 𝑎 + ) ⇒ 𝑐 − 𝑐 ≡ (𝑚𝑜𝑑 𝑎 + ) ⇔ (𝑎 + )|𝑐(𝑐 − 1)(𝑐 + 1) Cho 𝑛 → +∞ ta dễ thấy 𝑐(𝑐 − 1)(𝑐 + 1) = Từ suy 𝑐 = 𝑐 = - Trường hợp 1: 𝑐 = Nếu 𝑎 = theo đề |𝑏 ⇒ 4|𝑏 thoả điều kiện đề Nếu |𝑎| ≠ ta chứng minh tập ước nguyên tố {𝑎 + , 𝑛 ∈ ℤ } vô hạn Mà 𝑎 + |𝑏 ⇒ 𝑏 có vơ hạn ước số nguyên tố (vô lý) - Trường hợp 2: 𝑐 = Nếu 𝑎 chẵn theo đề 𝑎 + |𝑏 + ⇒ 𝑏 + ≡ (𝑚𝑜𝑑 4) (vô lý) Nếu 𝑎 lẻ, 2𝑎𝑏 không số phương nên 2𝑎 = 𝑙 𝑝 𝑝 … 𝑝 𝑏=𝑚 𝑝 𝑝 …𝑝 𝑏=𝑚 Với 𝑙, 𝑚, 𝑡, 𝑘 ∈ ℤ ; 𝑝 , 𝑝 , … , 𝑝 ∈ ℘; 𝑠 ∈ ℕ, số 𝑝 (𝑖 = 1, 𝑡 + 𝑠) phân biệt 𝑘 = 1, 𝑠 = vô lý 2𝑎𝑏 số phương Theo định lý thặng dư Trung Hoa tồn số nguyên tố 𝑝 cho 𝑝 ≡ (𝑚𝑜𝑑 4) Với 𝑏 = 𝑚 ⇒ = ∀𝑖 = 2, 𝑡; Với 𝑘 > 𝑡 ⇒ = −1 = ∀𝑖 = 𝑘, 𝑡 + 𝑠 = ∀𝑖 = 2, 𝑡; Với < 𝑘 ≤ 𝑡 ⇒ Với 𝑘 = 1, 𝑠 > ⇒ = ∀𝑖 = 2, 𝑡 + 𝑠, = −1 = ∀𝑖 = 2, 𝑡 + 𝑠 − Qua trường hợp ta thấy =1 = −1, = = −1 = nên theo tiêu chuẩn Euler ta có: (2𝑎) Mà 𝑎 = −1 (𝑚𝑜𝑑 𝑝) ⇒ 𝑎 +2 |𝑏 +1⇒𝑏 +2 ≡ (𝑚𝑜𝑑 𝑝); 𝑏 = (𝑚𝑜𝑑 𝑝) ≡ −1 (𝑚𝑜𝑑 𝑝) (mâu thuẫn 𝑝 ≠ 2) Vậy (𝑎, 𝑏, 𝑐) = (2,4𝑘, 0) với 𝑘 ∈ ℤ 𝑘 khơng số phương thoả mãn yêu cầu đề BÀI TẬP Bài 1: Chứng minh phương trình 3𝑥 + = 𝑦 khơng có nghiệm nguyên Bài 2: Chứng minh tồn dãy tăng {𝑎 } số tự nhiên cho ∀𝑘 ≥ 0, dãy {𝑘 + 𝑎 } chứa hữu hạn số nguyên tố Bài 3: Tìm tất số tự nhiên 𝑛 cho 2𝑛 − chia hết cho tồn 𝑚 ∈ ℤ cho 4𝑚 + chia hết cho Bài 4: Tìm tất thứ tự (𝑎, 𝑏, 𝑐, 𝑎 , 𝑏 , 𝑐 ) thoả 𝑎𝑏 + 𝑎 𝑏 ≡ (𝑚𝑜𝑑 15) 𝑏𝑐 + 𝑏 𝑐′ ≡ (𝑚𝑜𝑑 15) 𝑐𝑎 + 𝑐 𝑎 ≡ (𝑚𝑜𝑑 15) Với 𝑎, 𝑏, 𝑐, 𝑎 , 𝑏 , 𝑐 ∈ {0,1, … ,14} Bài 5: Cho hai số nguyên dương 𝑝, 𝑞 lớn nguyên tố Chứng minh tồn số nguyên 𝑘 cho (𝑝𝑞 − 1)𝑛𝑘 + hợp số với số nguyên dương 𝑛 Bài 6: Xét đa thức 𝑃(𝑥) = (2𝑥 + 1)(3𝑥 + 1) Chứng minh với 𝑛 nguyên dương, tồn 𝑥 nguyên cho 𝑃(𝑥) chia hết cho 𝑛 Bài 7: Cho 𝑝 số nguyên tố, ta có hệ phương trình đồng dư 𝑥 ≡ 𝑎 (𝑚𝑜𝑑 𝑚 ) 𝑥 ≡ 𝑎 (𝑚𝑜𝑑 𝑚 ) … 𝑥 ≡ 𝑎 (𝑚𝑜𝑑 𝑚 ) Và giả sử 𝑚 = 𝑝 , 𝑗 = 1, 𝑟 với ≤ 𝛼 ≤ 𝛼 ≤ ⋯ ≤ 𝛼 Chứng minh hệ phương trình có nghiệm 𝛼 ≡ 𝛼 (𝑚𝑜𝑑 𝑝 ) với 𝑖 = 1,2, … , 𝑟 Bài 8: Chứng minh định lý Euler mở rộng: 𝑎 ℤ ≡𝑎 ( ) (𝑚𝑜𝑑 𝑚) với 𝑎 ∈ Bài 9: Tìm số nghiệm phương trình 𝑥 ≡ 𝑥 (𝑚𝑜𝑑 𝑚) với số nguyên dương 𝑚 Bài 10: Đặt 𝜓(𝑛) số số nguyên 𝑎 cho ≤ 𝑎 ≤ 𝑛 (𝑎, 𝑛) = 1, (𝑎 + 1, 𝑛) = Chứng minh 𝜓(𝑛) = 𝑛 ∏ | (1 − 2/𝑝) Với giá trị 𝑛 𝜓(𝑛) = Bài 11: Một số ngun dương 𝑘 có tính chất 𝑇(𝑚) với số nguyên dương 𝑎, tồn số nguyên dương 𝑛 cho + + ⋯ + 𝑛 ≡ 𝑎 (𝑚𝑜𝑑 𝑚) a) Tìm tất số ngun dương 𝑘 có tính chất 𝑇(20) b) Tìm số ngun dương 𝑘 nhỏ có tính chất 𝑇(20 ) Bài 12: Cho 𝑛 𝑘 số nguyên dương Chứng minh 𝑛 30 nguyên tố tồn số nguyên 𝑎 𝑏, số nguyên tố với 𝑛 cho 𝑎 − 𝑏 + 𝑘 chia hết cho 𝑛 TÀI LIỆU THAM KHẢO [1] An Introduction to The Theory of Numbers, Ivan Niven; Herbert S Zuckerman; Hugh L Montgomery [2] Định lý thặng dư Trung Hoa số ứng dụng, Nguyễn Duy Liên [3] Định lý thặng dư Trung Hoa, Trần Minh Hiền [4] MathScope.org [5] Number Theory – Scuptures, Examples and Problems, Titu Andreescu; Dorin Andrica ... Ivan Niven; Herbert S Zuckerman; Hugh L Montgomery [2] Định lý thặng dư Trung Hoa số ứng dụng, Nguyễn Duy Liên [3] Định lý thặng dư Trung Hoa, Trần Minh Hiền [4] MathScope.org [5] Number Theory...

Ngày đăng: 27/12/2022, 19:37

Tài liệu cùng người dùng

Tài liệu liên quan